REPOST*
I NEED HELP ON THESE THREE QUESTIONS*
The number line compares the numbers –6, 3, and 5.

1. Write an inequality that compares –6 and 3. Justify your inequality using the number line.

2. Write an inequality that compares 3 and 5.

3. Write an inequality that compares –6, 3, and 5.

Answers

Answer 1

#1

-6-3-5

Hence inequality will be

[tex]\\ \sf\longmapsto -6\leqslant 5[/tex]

#2.

3<5

But we need inequality

[tex]\\ \sf\longmapsto 3\leqslant 5[/tex]

#3

The inequality given by

[tex]\\ \sf\longmapsto -6\leqslant 3\leqslant 5[/tex]

Answer 2

Answer:

See below

Step-by-step explanation:

Given numbers:

-6, 3 and 5

If we add these onto number line, we'll see them in same order from left to right.

We know the numbers increase from left to right on the number line, so the numbers to the left are smaller than the numbers to the right.

1.

-6 < 3, as -6 is to left from 3

2.

3 < 5, as 3 is to left from 5

3.

-6 < 3 < 5, as per above explanation

Related Questions

In ΔRST, r = 850 cm, s = 250 cm and t=940 cm. Find the measure of ∠S to the nearest 10th of a degree.

Answers

Answer:

∠s=15.0 *

Step-by-step explanation:

Help please!! i will give brainliest

Answers

Answer:

[tex]for \: - 1 \leqslant x < 1 \: y = - 3 \\ for \: \: 1 \leqslant x < 3 \: \: y = 1 \\ for \: 3 \leqslant x \leqslant 5 \: y = 5[/tex]

Taylor multiplied 8.25 * 10^4 and 0.4 * 10^4 and got 8.65 * 10^4. Explain Taylor's mistake. SOME ONE PLEASE HELP ME GET THIS ANSWER!

Answers

Answer: I believe the answer is 3.3 x 10^8

Somebody explain!!!!!!!!!!!

Answers

Answer:

$190

Step-by-step explanation:

Let's call the original price of the jacket "x".

If the new cost of the jacket is $87, we can represent this as x/2-8=87. By solving for x, we see that the original price of the jacket is $190

Answer:

I think the answer would be 182

Step-by-step explanation:

87x2= 174

174+8= 182

Pls help mee with this

Answers

The water will rise 10 in each hour after 4 hours the water will be 40 in deep and it will take 8 hours for the water to reach a depth of 80 in

Step-by-step explanation: POV: your a giga chad!

Which expression is equivalent to 10s+s-3s10s+s−3s?

Answers

Answer:

9s-3s10s or 3-10s

Step-by-step explanation:

10s+s-3s10s+s-3s

first of all, we collect like terms

10s+s+s-3s-3s10s

=9s-3s10s

it could also be equal to

9s/3s-(3s10s/3s)

3-10s

when we divide through by 3s

Plz help it’s due tonight!!!

Answers

LKM≈TEJ since it makes the triangles congruent, although the angles in each triangle are not corresponding equal angles

jeff decides to go to an outdoor skating rink the rink charges to rent a pair of skates and every hour of skating costs 2.00$ per person jeff. has at most 20$ to spend and needs to rent a pair of skates

Answers

Answer: The most amount he can rent the skates are 10 hours

Step-by-step explanation: if you divide the 2 by 20 it will be 10

The answer would be 72$ because u don’t add u subtract

The residence of a city voted on whether to raise property taxes the ratio of yes votes to no votes was 3 to 2 if there were 4440 total votes how many yes votes were there

Answers

Answer:

Y / N = 3 / 2

N + Y = 4440

Since Y = 3/2 N

3/2 N + N = 4440

5/2 N = 4440

N = 8880 / 5 = 1776

Y = 1776 * 3 / 2 = 2664

1776 + 2664 = 4440

2664  / 1776 = 3 / 2

−4 + 20 > −8 what the answers

Answers

Answer: -20

Step-by-step explanation: plz mark me brainliest please?

9/2 divided by 2 = 9/2 x2

True or false

Answers

Answer:

false

Step-by-step explanation:

the answer would be false

if f(x)=3x-1 and g(x)=x+2,find (f+g)(x)

Answers

Answer:

(f + g)(x) = 4x + 1

Step-by-step explanation:

Given f(x) = 3x - 1 and g(x) = x + 2, find (f + g)(x):

We can rewrite (f + g)(x) as f(x) + g(x), and solve the composite functions through addition:

f(x) + g(x) = (3x - 1) + (x + 2)

Combine like terms:

f(x) + g(x) = 3x + x + 2 - 1 = 4x + 1

Therefore, (f + g)(x) = 4x + 1.

Please mark my answers as the Brainliest if you find my explanations/solution helpful :)

Find the value of x.

Answers

Step-by-step explanation:

By using angle sum property

x-43+x-39+2x-18=180

4x=180+100

4x=280

x=280/4

x=70°

7x - 3 > 18 or 3x – 25-2

Answers

Hi I hope you are having a great day :) Your question, 7x - 3 > 18 or 3x – 25-2,

Let's solve your inequality step-by-step.

7x−3>(18)(3)x−25−2

1.  Simplify both sides of the inequality.

7x−3>54x−27

2.  Subtract 54x from both sides.

7x−3−54x>54x−27−54x

−47x−3>−27

3.  Add 3 to both sides.

−47x−3+3>−27+3

−47x>−24

4.  Divide both sides by -47

Finally answer, x<24/47

Hopefully that helps you :)

How do you choose the best point of view for your own narratives? A. You choose randomly. B. You think about how your choice of narrator will affect the story before you begin writing.C. You don't have to make a choice because it doesn't matter. D .You ask your teacher or a friend which point of view you should use.

Answers

Answer:

B.) You think about how your choice of narrator will affect the story before you begin writing, should be the most correct choice given. I hope this helps.

When the exponent is 2 and the base is unknown and the answer is 0.64 What is the base?

Answers

the base is 0.8
0.8^2 would also be 0.8 x 0.8
which will equal 0.64

Evaluate: 7.921 x 104.

79,210,000

7,921

792.1

79,210 Will mark brainly
Need now!!

Answers

Answer:

79,210,000

Step-by-step explanation:

:)

Find the slope of the line that passes through (6, 3) and (9, 5).

Answers

Answer:

2/3

Step-by-step explanation:

y2 - y1 / x2 - x1

5 - 3 / 9 - 6

2/3

What quantities are being compared? (If there are x- and y- axes, what do they represent?)

Answers

Answer:

The independent variable belongs on the x-axis (horizontal line) of the graph and the dependent variable belongs on the y-axis (vertical line).

WILL GIVE BRAINLIEST TO FIRST PERSON WHO ACTUALLY TRIES!!!

What is the length of the unknown leg of the right triangle rounded to the nearest tenth of a foot?​

Answers

Answer:

Around 8.8

Step-by-step explanation:

This problem requires the Pythagorean Theorem. We are given the hypotenuse and a leg, so we can plug in what we know.

[tex]{x^2+2^2}=9^2 \\{4+x^2}=81 \\x^2=77\\\sqrt{x^2}=77\\x=\sqrt{77}\\x=8.8, approximately[/tex]

5. For which value of x is line 1 parallel to line m? (1 point)

Answers

Answer: 62

Step-by-step explanation: you set up the equation by making the two sums equal eachother. 2x+6=130. then you solve, subtract six from both sides, then you have 2x=124. divide both sides by 2 to get the sum of x. you get x=62

Need to finish this in 5 mins

Answers

Answer:

Paper is really blurry sorry, cant see.

Step-by-step explanation:

Need help ASAP! Directions in picture ;)

Answers

h= 92 and h and f equal the same cuz they’re congruent (i think is the right term) so 5x+4=92 so x would be 17.6

find the size of deb

Answers

Answer:

[tex]4x + 3 = 6x - 53 \\ 6x - 4x = 3 + 53 \\ 2x = 56 \\ x = \frac{56}{2} = 28[/tex]

what is the midpoint of (0,0.55) and (-3,6)

Answers

Answer:

(-1.5, 3.275)

Step-by-step explanation:

midpoint = ( x1+X2/2 , y1+y2/2)

=(0+(-3)/2, 0.55+6/2)

=( -3/2, 6.55/2)

=( -1.5, 3.275)

hola alguien para hablar estoy aburrida y me gustaría hablar con alguien

Answers

Answer:

i know english only

Step-by-step explanation:

sorry but you stupit you not good at englis

Round each number to the nearest thousand.

1,239 and 8,568

1,000 and 9,000

1,000 and 8,000

2,000 and 8,000

2.000 and 9.000

Answers

It would be the first option- 1,000 and 9,000

solve the inequality. 1/3b + 1 < -6​

Answers

Answer:

b<-21

Step-by-step explanation:

1/3b+1<-6

1/3b<-7

*3     *3

b<-21

Answer:

b <  - 21

Step-by-step explanation:

[tex]\frac{1}{3}[/tex] b + 1 < - 6 ( subtract 1 from both sides )

[tex]\frac{1}{3}[/tex] b < - 7 ( multiply both sides by 3 to clear the fraction )

b < - 21

Find the x and y intercepts of this function.

h(x) = 2x^2 - 3x - 2

Please help me

Answers

Answer:

The x-intercepts: (-0.5, 0) and (2, 0)

The y-intercept: (0, -2)

Between which two consecutive whole numbers can we find 96−−√?

A.7 and 8
B.8 and 9
C.9 and 10

Answers

C. 9 and 10 I think


Hoped it helped (:
Other Questions